Quoc Tran Anh Le
Cho hai hàm số fleft( x right) {x^2} - 1,gleft( x right) x + 1.a) Tính mathop {lim }limits_{x to 1} fleft( x right) và mathop {lim }limits_{x to 1} gleft( x right).b) Tính mathop {lim }limits_{x to 1} left[ {fleft( x right) + gleft( x right)} right]và so sánh mathop {lim }limits_{x to 1} fleft( x right) + mathop {lim }limits_{x to 1} gleft( x right).c) Tính mathop {lim }limits_{x to 1} left[ {fleft( x right) - gleft( x right)} right]và so sánh mathop {lim }limits_{x to 1} fleft( x right) - mat...
Đọc tiếp

Những câu hỏi liên quan
Quoc Tran Anh Le
Xem chi tiết
Hà Quang Minh
22 tháng 9 2023 lúc 21:19

a) \(\mathop {\lim }\limits_{x \to 1} f\left( x \right) = \mathop {\lim }\limits_{x \to 1} x = 1\)

b) \(f\left( 1 \right) = 1 \Rightarrow \mathop {\lim }\limits_{x \to 1} f\left( x \right) = f\left( 1 \right).\)

Bình luận (0)
Buddy
Xem chi tiết
Hà Quang Minh
22 tháng 9 2023 lúc 12:11

\(\mathop {\lim }\limits_{x \to {1^ + }} f\left( x \right) = \mathop {\lim }\limits_{x \to {1^ + }} x = 1\).

\(\mathop {\lim }\limits_{x \to {1^ - }} f\left( x \right) = \mathop {\lim }\limits_{x \to {1^ - }} \left( { - {x^2}} \right) =  - {1^2} =  - 1\).

Vì \(\mathop {\lim }\limits_{x \to {1^ + }} f\left( x \right) \ne \mathop {\lim }\limits_{x \to {1^ - }} {\rm{ }}f\left( x \right)\) nên không tồn tại \(\mathop {\lim }\limits_{x \to 1} f\left( x \right)\).

Bình luận (0)
Quoc Tran Anh Le
Xem chi tiết
Hà Quang Minh
22 tháng 9 2023 lúc 21:17

Vì \(\mathop {\lim }\limits_{x \to {2^ - }} f\left( x \right) = 3 \ne \mathop {\lim }\limits_{x \to {2^ + }} f\left( x \right) = 5\) nên không tồn tại giới hạn \(\mathop {\lim }\limits_{x \to 2} f\left( x \right)\)

Bình luận (0)
Buddy
Xem chi tiết
Quoc Tran Anh Le
22 tháng 9 2023 lúc 11:42

a) Giả sử \(\left( {{x_n}} \right)\) là dãy số bất kì, \({x_n} >  - 1\) và \({x_n} \to  - 1\). Khi đó \(f\left( {{x_n}} \right) = x_n^2 + 2\)

Ta có: \(\lim f\left( {{x_n}} \right) = \lim \left( {x_n^2 + 2} \right) = \lim x_n^2 + \lim 2 = {\left( { - 1} \right)^2} + 2 = 3\)

Vậy \(\mathop {\lim }\limits_{x \to  - {1^ + }} f\left( x \right) = 3\).

Giả sử \(\left( {{x_n}} \right)\) là dãy số bất kì, \({x_n} <  - 1\) và \({x_n} \to  - 1\). Khi đó \(f\left( {{x_n}} \right) = 1 - 2{x_n}\).

Ta có: \(\lim f\left( {{x_n}} \right) = \lim \left( {1 - 2{x_n}} \right) = \lim 1 - \lim \left( {2{x_n}} \right) = \lim 1 - 2\lim {x_n} = 1 - 2.\left( { - 1} \right) = 3\)

Vậy \(\mathop {\lim }\limits_{x \to  - {1^ - }} f\left( x \right) = 3\).

b) Vì \(\mathop {\lim }\limits_{x \to  - {1^ + }} f\left( x \right) = \mathop {\lim }\limits_{x \to  - {1^ - }} {\rm{ }}f\left( x \right) = 3\) nên \(\mathop {\lim }\limits_{x \to  - 1} f\left( x \right) = 3\).

Bình luận (0)
Quoc Tran Anh Le
Xem chi tiết
Hà Quang Minh
22 tháng 9 2023 lúc 15:56

a) \(\mathop {\lim }\limits_{x \to 2} \left[ {\left( {x + 1} \right)\left( {{x^2} + 2x} \right)} \right] = \mathop {\lim }\limits_{x \to 2} \left( {x + 1} \right).\mathop {\lim }\limits_{x \to 2} \left( {{x^2} + 2x} \right) = \left( {2 + 1} \right).\left( {{2^2} + 2.2} \right) = 24\)                

b) \(\mathop {\lim }\limits_{x \to 2} \sqrt {{x^2} + x + 3}  = \sqrt {\mathop {\lim }\limits_{x \to 2} \left( {{x^2} + x + 3} \right)}  = \sqrt {\mathop {\lim }\limits_{x \to 2} {x^2} + \mathop {\lim }\limits_{x \to 2} x + \mathop {\lim }\limits_{x \to 2} 3}  = \sqrt {{2^2} + 2 + 3}  = 3\)

Bình luận (0)
Buddy
Xem chi tiết
Hà Quang Minh
22 tháng 9 2023 lúc 21:07

a) Ta có \(t = \frac{1}{x},\) nên khi x tiến đến 0 thì t tiến đến dương vô cùng do đó

\(\mathop {\lim }\limits_{x \to 0} {\left( {1 + x} \right)^{\frac{1}{x}}} = \mathop {\lim }\limits_{t \to  + \infty } {\left( {1 + \frac{1}{t}} \right)^t} = e\)

b) \(\ln y = \ln {\left( {1 + x} \right)^{\frac{1}{x}}} = \frac{1}{x}\ln \left( {1 + x} \right)\)

\(\mathop {\lim }\limits_{x \to 0} \ln y = \mathop {\lim }\limits_{x \to 0} \frac{{\ln \left( {1 + x} \right)}}{x} = 1\)

c) \(t = {e^x} - 1 \Leftrightarrow {e^x} = t + 1 \Leftrightarrow x = \ln \left( {t + 1} \right)\)

\(\mathop {\lim }\limits_{x \to 0} \frac{{{e^x} - 1}}{x} = \mathop {\lim }\limits_{t \to 0} \frac{t}{{\ln \left( {t + 1} \right)}} = 1\)

Bình luận (0)
Buddy
Xem chi tiết
Nguyễn Lê Phước Thịnh
23 tháng 7 2023 lúc 11:19

a: \(\lim\limits_{x\rightarrow-1^+}x+1=0\)

=>\(\lim\limits_{x\rightarrow-1^+}\dfrac{1}{x+1}=+\infty\)

b: \(\lim\limits_{x\rightarrow-\infty}1-x^2=\lim\limits_{x\rightarrow-\infty}\left[x^2\left(\dfrac{1}{x^2}-1\right)\right]\)

\(=-\infty\)

c: \(\lim\limits_{x\rightarrow3^-}\dfrac{x}{3-x}=\lim\limits_{x\rightarrow3^-}=\dfrac{-x}{x-3}\)

\(\lim\limits_{x\rightarrow3^-}x-3=0\)

\(\lim\limits_{x\rightarrow3^-}-x=3>0\)

=>\(\lim\limits_{x\rightarrow3^-}\dfrac{x}{3-x}=+\infty\)

Bình luận (0)
Quoc Tran Anh Le
Xem chi tiết
Hà Quang Minh
22 tháng 9 2023 lúc 21:18

a) \(\mathop {\lim }\limits_{x \to 2} \left( {{x^2} - 4x + 3} \right) = \mathop {\lim }\limits_{x \to 2} {x^2} - \mathop {\lim }\limits_{x \to 2} \left( {4x} \right) + 3 = {2^2} - 4.2 + 3 =  - 1\)

b) \(\mathop {\lim }\limits_{x \to 3} \frac{{{x^2} - 5x + 6}}{{x - 3}} = \mathop {\lim }\limits_{x \to 3} \frac{{\left( {x - 3} \right)\left( {x - 2} \right)}}{{x - 3}} = \mathop {\lim }\limits_{x \to 3} \left( {x - 2} \right) = \mathop {\lim }\limits_{x \to 3} x - 2 = 3 - 2 = 1\)

c) \(\mathop {\lim }\limits_{x \to 1} \frac{{\sqrt x  - 1}}{{x - 1}} = \mathop {\lim }\limits_{x \to 1} \frac{{\sqrt x  - 1}}{{\left( {\sqrt x  - 1} \right)\left( {\sqrt x  + 1} \right)}} = \mathop {\lim }\limits_{x \to 1} \frac{1}{{\sqrt x  + 1}} = \frac{1}{{\sqrt 1  + 1}} = \frac{1}{2}\)

Bình luận (0)
Buddy
Xem chi tiết
Quoc Tran Anh Le
22 tháng 9 2023 lúc 11:39

a) Đặt \(f\left( x \right) = 2{x^2} - x\).

Hàm số \(y = f\left( x \right)\) xác định trên \(\mathbb{R}\).

Giả sử \(\left( {{x_n}} \right)\) là dãy số bất kì thỏa mãn \({x_n} \to 3\) khi \(n \to  + \infty \). Ta có:

\(\lim f\left( {{x_n}} \right) = \lim \left( {2x_n^2 - {x_n}} \right) = 2.\lim x_n^2 - \lim {x_n} = {2.3^2} - 3 = 15\).

Vậy \(\mathop {\lim }\limits_{x \to 3} \left( {2{x^2} - x} \right) = 15\).

b) Đặt \(f\left( x \right) = \frac{{{x^2} + 2x + 1}}{{x + 1}}\).

Hàm số \(y = f\left( x \right)\) xác định trên \(\mathbb{R}\).

Giả sử \(\left( {{x_n}} \right)\) là dãy số bất kì thỏa mãn \({x_n} \to  - 1\) khi \(n \to  + \infty \). Ta có:

\(\lim f\left( {{x_n}} \right) = \lim \frac{{x_n^2 + 2{x_n} + 1}}{{{x_n} + 1}} = \lim \frac{{{{\left( {{x_n} + 1} \right)}^2}}}{{{x_n} + 1}} = \lim \left( {{x_n} + 1} \right) = \lim {x_n} + 1 =  - 1 + 1 = 0\).

Vậy \(\mathop {\lim }\limits_{x \to  - 1} \frac{{{x^2} + 2x + 1}}{{x + 1}} = 0\).

Bình luận (0)